1
$\begingroup$

Let $a,b\in{\mathbb Z}^{\ge0}$ and $h\in{\mathbb R}(d_1,d_2)$ be such that $$a\ge b, \quad h(d_1,d_2)>0~~\forall\,d_1,d_2\in{\mathbb Z}^+, \quad \lim_{d_1,d_2\longrightarrow\infty}\frac{h(d_1,d_2)}{d_1^ad_2^a/(d_1\!+\!d_2)^b}\in{\mathbb R}^+\,.$$ The limit statement means that these fractions are within $\epsilon$ of some number for all $d_1,d_2\!\ge\!d(\epsilon)$.

For $d\!=\!1,2,\ldots$, define the numbers $n_d\!\in\!{\mathbb R}^+$ inductively by $$n_1=1, \qquad n_d=\sum_{\begin{subarray}{c}d_1+d_2=d\\ d_1,d_2\ge1\end{subarray}}\!\!\!\! h(d_1,d_2)n_{d_1}n_{d_2} \quad\forall~d>1.$$ Are the numbers $\sqrt[d]{n_d}$ eventually increasing? In other words, is there $d^*\!\in\!{\mathbb Z}^+$ such that $$\sqrt[d]{n_d}\le\sqrt[d+1]{n_{d+1}} \qquad\forall~ d\ge d^*.$$ This is true if $h(d_1,d_2)=d_1^a d_2^a/(d_1\!+\!d_2)^a$. In general, these roots are bounded above and below away from zero.

$\endgroup$
9
  • $\begingroup$ Dear Aleksey, Could you say a little more about what this has to do with the recursion relations from Gromov-Witten theory? That might make it easier to put this question in context. $\endgroup$ Mar 28, 2016 at 12:56
  • $\begingroup$ I am sure that you know this already, but you may as well replace the initial condition, $n_1=1$, by the condition that $n_1>0$. The set of solutions is invariant under the transformation $n_d^{\text{new}} = a^d n_d^{\text{old}}$ for $a>0$. Also the conclusion you want to prove is invariant under this transformation. $\endgroup$ Mar 28, 2016 at 15:13
  • $\begingroup$ Okay, here is another silly observation. Assume that $a>0$. By your hypothesis, there exists $d(\epsilon)$ such that for all $d \geq d(\epsilon)$, also $h(d,d) \geq d^{2a-b}/u$ for some $u>0$. Then, for every $v>1$, there is a larger integer $d^*$ such that for all $d\geq d^*$, also $d^{2a-b}/u >v$. Then $\sqrt{2d}{n_{2d}} \geq v\cdot \sqrt{d}{n_d}$. Thus, even if the sequence $(\sqrt{d}{n_d})$ is not increasing, nonetheless it has many increasing subsequences. $\endgroup$ Mar 28, 2016 at 15:51
  • $\begingroup$ Typo correction: The LaTeX was bad for the final 2 sentences. It should have read "Then $\sqrt[2d]{n_{2d}} \geq \sqrt[d]{n_d}$. Thus, even if the sequence $(\sqrt[d]{n_d})_d$ is not increasing, nonetheless it has many increasing subsequences." $\endgroup$ Mar 28, 2016 at 16:27
  • $\begingroup$ In your applications, do you have any a priori upper bounds on $h(d_1,d_2)$ for $d_1+d_2 < 2d(\epsilon)$? If so, you could try to make my previous remark into an induction proof. $\endgroup$ Mar 28, 2016 at 16:57

0

Your Answer

By clicking “Post Your Answer”, you agree to our terms of service and acknowledge you have read our privacy policy.

Browse other questions tagged or ask your own question.